Inscription / Connexion Nouveau Sujet
Niveau autre
Partager :

probeleme integrale

Posté par pilote7 (invité) 04-06-05 à 23:36

Bonjour,
je suis pas tres doué pour la résolution d'integrale mais celle la encore moins.
Pouvez vous m'aider?
Merci d'avance
(x+2)arctan(((x+3)/(x+1)))dx entre 1 et 2

Posté par
Nightmare
re : probeleme integrale 04-06-05 à 23:53

Alors moi je propose le changement de variable :

3$\rm u=-4 \(-\frac{x+3}{(x+1)\(\frac{2(x+3)^{2}}{(x-1)^{2}}-\frac{4(x+3)}{x+1}+2\)}arctan\(\sqrt{\frac{x+3}{x+1}}\)-\frac{1}{8}arctan\(\sqrt{x+3}{x+1}\)-\frac{1}{\frac{8(x+3)}{x+1}-8}\sqrt{\frac{x+3}{x+1}}\)

On obtient :
5$\rm \Bigint (x+2)arctan\(\sqrt{\frac{x+3}{x+1}}\)dx=\Bigint du
soit :
5$\rm  \Bigint (x+2)arctan\(\sqrt{\frac{x+3}{x+1}}\)dx=u
c'est à dire :
3$\rm\Bigint (x+2)arctan\(\sqrt{\frac{x+3}{x+1}}\)dx=-4 \(-\frac{x+3}{(x+1)\(\frac{2(x+3)^{2}}{(x-1)^{2}}-\frac{4(x+3)}{x+1}+2\)}arctan\(\sqrt{\frac{x+3}{x+1}}\)-\frac{1}{8}arctan\(\sqrt{x+3}{x+1}\)-\frac{1}{\frac{8(x+3)}{x+1}-8}\sqrt{\frac{x+3}{x+1}}\)+C




Jord

Posté par pilote7 (invité)re : probeleme integrale 05-06-05 à 13:08

salut,
je vois pas trop comment en peut la calculer car on donne quoi comme valeur a "C"?
Moi j'ai commence par faire une integration par partie mais j'arrive pas au bout.
Merci de m'aider

Posté par
otto
re : probeleme integrale 05-06-05 à 13:39

Tu donnes bien la valeur que tu veux, de toute facon lorsque tu la calcules entre 1 et 2 elle va sauter.
A+

Posté par
J-P Posteur d'énigmes
re : probeleme integrale 05-06-05 à 14:04

Attention, Nightmare a visiblement triché pour résoudre cette intégrale.
Il a fait résoudre la primitive par un logiciel quelconque et ensuite posé la solution trouvée par ce logiciel comme changement de variable.

Il l'a montré par les 3 smileys en fin de son message, mais cela ne semblait pas clair pour tout le monde.


Posté par
Nightmare
re : probeleme integrale 05-06-05 à 14:10

Lol effectivement J-P , tu m'as démasqué

Moi j'aime bien cette technique , à condition d'avoir un logiciel de calcul de primitive ou une calculette à côté


jord

Posté par
dad97 Correcteur
re : probeleme integrale 05-06-05 à 14:14

"Alors moi je propose le changement de variable : " le moi n'est peut être pas adapté (wims )

et quand on recopie (ouh c'est pas bien) on ne fait pas d'erreur

on ne divise pas par (x-1)² mais par (x+1)² :

3$\rm\Bigint_1^2 (x+2)arctan\(\sqrt{\frac{x+3}{x+1}}\)dx
3$\rm = 4 \[-\frac{x+3}{(x+1)\(\frac{2(x+3)^{2}}{(x+1)^{2}} + \frac{4(x+3)}{x+1}+2\)}arctan\(\sqrt{\frac{x+3}{x+1}}\) + \frac{1}{8}arctan(\sqrt{\frac{x+3}{x+1}})+\frac{1}{\frac{8(x+3)}{x+1}-8} \sqrt{\frac{x+3}{x+1}}\]_1^2

Salut

Posté par
Nightmare
re : probeleme integrale 05-06-05 à 14:18

Lol merci Dad97 , c'est vrai que je n'ai pas pensé à relire mon truc abominable


Jord

Posté par
Redman
re : probeleme integrale 05-06-05 à 14:19

c'est quel niveau ca? prépa?

Posté par
Nightmare
re : probeleme integrale 05-06-05 à 14:27

Oui , c'est abordable en sup je pense


Jord

Posté par
otto
re : probeleme integrale 05-06-05 à 15:16

La meilleure idée reste l'IPP suivi d'un développement en éléments simples.
L'ipp va faire "sauter" l'arctan.
A+

Posté par pilote7 (invité)re : probeleme integrale 05-06-05 à 15:51

salut a tous et merci pour vos reponses.
Cependant j'ai vraiment du mal, j'ai fait l'integration par partie j'obtient:
u=arctan(((x+3)/(x+1))
v'=(x+2)
v=(x2/2)+2x+c
u'=1/(1+(((x+3)/(x+1)))2)=1/(1+(x+3)/(x+1))

donc =[(x2/2+2x+C)(arctan(((x+3)/(x+1)))]-(1/(1+(x+3)/(x+1)))(x+2))dx

est ce le bon resultat?
Merci d'avance

Posté par
Nightmare
re : probeleme integrale 05-06-05 à 16:16

Re pilote7

Je ne suis pas daccord avec ta dérivée de l'arctangente

Pour toute fonction u dérivable:
3$\rm (arctan(u))'=\frac{u'}{1+u^{2}} (dérivé de fonction composée)


jord

Posté par
dad97 Correcteur
re : probeleme integrale 05-06-05 à 16:19

Allons y pour une IPP :

on pose 3$\rm u(x)=arctan(\sqrt{\frac{x+3}{x+1}}) donc 3$\rm u^'(x)=\frac{-1}{2(x+2)\sqrt{x+1}\sqrt{x+3}}

et 3$\rm v^'(x)=x+2 donc 3$\rm v(x)=\frac{1}{2}(x+2)^2

d'où :

3$\rm \Bigint_1^2 (x+2)arctan\(\sqrt{\frac{x+3}{x+1}}\)dx=[\frac{1}{2}(x+2)^2\times arctan(\sqrt{\frac{x+3}{x+1}})]_1^2+\frac{1}{4}\times\Bigint_1^2\frac{x+2}{\sqrt{x+1}\sqrt{x+3}}dx

or 3$\rm \Bigint_1^2\frac{x+2}{\sqrt{x+1}\sqrt{x+3}}dx=\Bigint_1^2 \frac{2(x+2)}{2\sqrt{x^2+4x+3}}=[\sqrt{x^2+4x+3}]_1^2

d'où
3$\rm \Bigint_1^2 (x+2)arctan\(\sqrt{\frac{x+3}{x+1}}\)dx=[\frac{1}{2}(x+2)^2\times arctan(\sqrt{\frac{x+3}{x+1}})+\frac{1}{4}\sqrt{x^2+4x+3}]_1^2

d'où :

4$\rm\blue\fbox{ \Bigint_1^2 (x+2)arctan\(\sqrt{\frac{x+3}{x+1}}\)dx=8Arctan(\sqrt{\frac{5}{3}})+\frac{\sqrt{15}}{4}-\frac{9}{2}Arctan(\sqrt{2})-\frac{\sqrt{2}}{2}
Salut

Posté par pilote7 (invité)re : probeleme integrale 06-06-05 à 00:39

merci pour vos reponse mais le resultat donne par dad97 ne me semble pas correcte car j'ai approche l'integrale par la methode de monte carlo et j'obtient comme resultat 3.25.. alors qu'avec celle de dad97 je trouve 2.7230 sauf erreur de ma part.
merci de m'aider

Posté par
elhor_abdelali Correcteur
re : probeleme integrale 06-06-05 à 03:24

Commmençons par un petit changement de variable: u=x+2
on obtient alors si on note I l'intégrale à calculer:
4
I = u*arctan((u+1)/(u-1))du
3
procédons maintenant par une intégration par parties on a I=J+K où:
4
J = [(u²/2)*arctan((u+1)/(u-1))] = 8*arctan((5/3))-(9/2)*arctan(2)
3
4
K=(1/4)* udu/(u²-1)
3
K se calcule facilement par le changement de variable: u=ch(t) (cosinus hyperbolique)
on obtient:
argch(4)
K = ch(t)dt= sh(argch(4))-sh(argch(3))=15 - 8
argch(3)
finalement sauf erreur on a :
I = 8*arctan((5/3))-(9/2)*arctan(2)+15 - 8

signalons que l'on obtient aussi une primitive de x(x+2)*arctan((x+3)/(x+1)) à savoir la fonction:
x(1/2)*(x+2)²*arctan((x+3)/(x+1)) + (1/4)*(x²+4*x+3)

Posté par
elhor_abdelali Correcteur
re : probeleme integrale 06-06-05 à 03:40

je m'excuse j'ai oublié de multiplier par 1/4 dans l'expression de K on a alors:

I=8*arctan((5/3))-(9/2)*arctan(2)+15/4-2/2
le résultat de dad97 me semble correct

Posté par
lyonnais
re : probeleme integrale 06-06-05 à 09:59

salut pilote7 :

j'y comprend strictement rien à vos histoire de tan-1 , mais ce que je peux te dire, c'est que le résultat de dad97 est bon

En efet, j'ai un programme sur la calculatrice, et grâc à lui je trouve 3,2561

Ce qui est le même résultat que quans on fait le calcul de dad97 !

es-tu sur que ta calculatrice soit en radian ...

@+ sur l'

Posté par pilote7 (invité)re : probeleme integrale 06-06-05 à 10:10

mille excuses a dad97 je me suis trompe dans mes calculs.
Je savais qu'il na fallait pas travailler le dimanche
Merci a tout le monde pour vos reponses



Vous devez être membre accéder à ce service...

Pas encore inscrit ?

1 compte par personne, multi-compte interdit !

Ou identifiez-vous :


Rester sur la page

Inscription gratuite

Fiches en rapport

parmi 1675 fiches de maths

Désolé, votre version d'Internet Explorer est plus que périmée ! Merci de le mettre à jour ou de télécharger Firefox ou Google Chrome pour utiliser le site. Votre ordinateur vous remerciera !